Show that $(C([a,b]),|{cdot}|_2)$ is not a complete normed vector space












4












$begingroup$


Problem: Show that $(C([a,b]),|{cdot}|_2)$ is not a complete normed vector space.



I have tried to proceed along the lines given in the accepted answer, and subsequent comment, to this question: Example of a non complete normed vector space. Although, I am running into difficulties and wondering if the example really generalises so well when we have a general $[a,b]$. Is the Cauchy sequence I have chosen below a valid one, or have I overlooked a much simpler example to use in this case?



Attempt: In order to show that $(C([a,b]),|{cdot}|_2)$ is not a complete normed vector space we need to show that there is some Cauchy sequence that does not converge to it's limit in $C([a,b])$. In particular, it will suffice to show that our chosen Cauchy sequence converges under $|cdot|_2$ to a discontinuous function.



Consider the following sequence of functions, $(f_n)_{n=1}^inftysubset C([a,b])$, given by,



$$f_n(x)=begin{cases}
0,,text{when},,xin[a,frac{b-a}2-frac{2^{-n}}{b-a}] \[2ex]
1,,text{when},,xin[frac{b-a}2+frac{2^{-n}}{b-a},b] \[2ex]
frac{(2x-b+a)(b-a)-2^{-n+1}}{2^{-n+2}},,text{when},,xin[frac{b-a}2-frac{2^{-n}}{b-a},frac{b-a}2+frac{2^{-n}}{b-a}]
end{cases}
$$



The definition of $f_n(x)$ when $xin[frac{b-a}2-frac{2^{-n}}{b-a},frac{b-a}2+frac{2^{-n}}{b-a}]$ is the linear interpolant over that particular subinterval of $[a,b]$.



I claim that this is a Cauchy sequence. We consider:



$$|f_n-f_m|_2^2=int_a^b|f_n(x)-f_m(x)|^2dx$$



$$=int_a^{frac{b-a}2-frac{2^{-n}}{b-a}}|f_n(x)-f_m(x)|^2dx+int_{frac{b-a}2-frac{2^{-n}}{b-a}}^{frac{b-a}2+frac{2^{-n}}{b-a}}|f_n(x)-f_m(x)|^2dx+int_{frac{b-a}2+frac{2^{-n}}{b-a}}^b|f_n(x)-f_m(x)|^2dx$$



$$=0+int_{frac{b-a}2-frac{2^{-n}}{b-a}}^{frac{b-a}2+frac{2^{-n}}{b-a}}|f_n(x)-f_m(x)|^2dx+0$$



The $0$'s for the first and last integrals because we have $f_n(x)-f_m(x)=0-0$ and $f_n(x)-f_m(x)=1-1$ on each respectively. As to the middle integral, here is what I've gotten so far:



$$int_{frac{b-a}2-frac{2^{-n}}{b-a}}^{frac{b-a}2+frac{2^{-n}}{b-a}}|f_n(x)-f_m(x)|^2dx=int_{frac{b-a}2-frac{2^{-n}}{b-a}}^{frac{b-a}2+frac{2^{-n}}{b-a}}|(2x-b-a)(b-a)(2^{n-2}-2^{m-2})|^2dx$$



Based in the example I am following, supposing that $mge n$, I think that I should be able to argue that the whole thing is bound above by $frac{2^{-n}}{b-a}$ which would mean that $|f_n-f_m|_2^2lefrac{2^{-n}}{b-a}to0$ as $ntoinfty$.



I don't quite see how I'm meant to bound it above by $frac{2^{-n}}{b-a}$, as I keep tripping up over applying that $mge n$, leading me to question whether the approach I have employed here is a fruitful one after all.










share|cite|improve this question











$endgroup$












  • $begingroup$
    one of the functions dominates the other, so you just have to subtract the individual integrals. Can you integrate linear function?
    $endgroup$
    – Jorge Fernández
    Mar 14 '18 at 22:29








  • 1




    $begingroup$
    instead of $frac{b-a}{2}$ you should have $frac{b+a}{2}$
    $endgroup$
    – Orest Bucicovschi
    Mar 15 '18 at 1:58
















4












$begingroup$


Problem: Show that $(C([a,b]),|{cdot}|_2)$ is not a complete normed vector space.



I have tried to proceed along the lines given in the accepted answer, and subsequent comment, to this question: Example of a non complete normed vector space. Although, I am running into difficulties and wondering if the example really generalises so well when we have a general $[a,b]$. Is the Cauchy sequence I have chosen below a valid one, or have I overlooked a much simpler example to use in this case?



Attempt: In order to show that $(C([a,b]),|{cdot}|_2)$ is not a complete normed vector space we need to show that there is some Cauchy sequence that does not converge to it's limit in $C([a,b])$. In particular, it will suffice to show that our chosen Cauchy sequence converges under $|cdot|_2$ to a discontinuous function.



Consider the following sequence of functions, $(f_n)_{n=1}^inftysubset C([a,b])$, given by,



$$f_n(x)=begin{cases}
0,,text{when},,xin[a,frac{b-a}2-frac{2^{-n}}{b-a}] \[2ex]
1,,text{when},,xin[frac{b-a}2+frac{2^{-n}}{b-a},b] \[2ex]
frac{(2x-b+a)(b-a)-2^{-n+1}}{2^{-n+2}},,text{when},,xin[frac{b-a}2-frac{2^{-n}}{b-a},frac{b-a}2+frac{2^{-n}}{b-a}]
end{cases}
$$



The definition of $f_n(x)$ when $xin[frac{b-a}2-frac{2^{-n}}{b-a},frac{b-a}2+frac{2^{-n}}{b-a}]$ is the linear interpolant over that particular subinterval of $[a,b]$.



I claim that this is a Cauchy sequence. We consider:



$$|f_n-f_m|_2^2=int_a^b|f_n(x)-f_m(x)|^2dx$$



$$=int_a^{frac{b-a}2-frac{2^{-n}}{b-a}}|f_n(x)-f_m(x)|^2dx+int_{frac{b-a}2-frac{2^{-n}}{b-a}}^{frac{b-a}2+frac{2^{-n}}{b-a}}|f_n(x)-f_m(x)|^2dx+int_{frac{b-a}2+frac{2^{-n}}{b-a}}^b|f_n(x)-f_m(x)|^2dx$$



$$=0+int_{frac{b-a}2-frac{2^{-n}}{b-a}}^{frac{b-a}2+frac{2^{-n}}{b-a}}|f_n(x)-f_m(x)|^2dx+0$$



The $0$'s for the first and last integrals because we have $f_n(x)-f_m(x)=0-0$ and $f_n(x)-f_m(x)=1-1$ on each respectively. As to the middle integral, here is what I've gotten so far:



$$int_{frac{b-a}2-frac{2^{-n}}{b-a}}^{frac{b-a}2+frac{2^{-n}}{b-a}}|f_n(x)-f_m(x)|^2dx=int_{frac{b-a}2-frac{2^{-n}}{b-a}}^{frac{b-a}2+frac{2^{-n}}{b-a}}|(2x-b-a)(b-a)(2^{n-2}-2^{m-2})|^2dx$$



Based in the example I am following, supposing that $mge n$, I think that I should be able to argue that the whole thing is bound above by $frac{2^{-n}}{b-a}$ which would mean that $|f_n-f_m|_2^2lefrac{2^{-n}}{b-a}to0$ as $ntoinfty$.



I don't quite see how I'm meant to bound it above by $frac{2^{-n}}{b-a}$, as I keep tripping up over applying that $mge n$, leading me to question whether the approach I have employed here is a fruitful one after all.










share|cite|improve this question











$endgroup$












  • $begingroup$
    one of the functions dominates the other, so you just have to subtract the individual integrals. Can you integrate linear function?
    $endgroup$
    – Jorge Fernández
    Mar 14 '18 at 22:29








  • 1




    $begingroup$
    instead of $frac{b-a}{2}$ you should have $frac{b+a}{2}$
    $endgroup$
    – Orest Bucicovschi
    Mar 15 '18 at 1:58














4












4








4





$begingroup$


Problem: Show that $(C([a,b]),|{cdot}|_2)$ is not a complete normed vector space.



I have tried to proceed along the lines given in the accepted answer, and subsequent comment, to this question: Example of a non complete normed vector space. Although, I am running into difficulties and wondering if the example really generalises so well when we have a general $[a,b]$. Is the Cauchy sequence I have chosen below a valid one, or have I overlooked a much simpler example to use in this case?



Attempt: In order to show that $(C([a,b]),|{cdot}|_2)$ is not a complete normed vector space we need to show that there is some Cauchy sequence that does not converge to it's limit in $C([a,b])$. In particular, it will suffice to show that our chosen Cauchy sequence converges under $|cdot|_2$ to a discontinuous function.



Consider the following sequence of functions, $(f_n)_{n=1}^inftysubset C([a,b])$, given by,



$$f_n(x)=begin{cases}
0,,text{when},,xin[a,frac{b-a}2-frac{2^{-n}}{b-a}] \[2ex]
1,,text{when},,xin[frac{b-a}2+frac{2^{-n}}{b-a},b] \[2ex]
frac{(2x-b+a)(b-a)-2^{-n+1}}{2^{-n+2}},,text{when},,xin[frac{b-a}2-frac{2^{-n}}{b-a},frac{b-a}2+frac{2^{-n}}{b-a}]
end{cases}
$$



The definition of $f_n(x)$ when $xin[frac{b-a}2-frac{2^{-n}}{b-a},frac{b-a}2+frac{2^{-n}}{b-a}]$ is the linear interpolant over that particular subinterval of $[a,b]$.



I claim that this is a Cauchy sequence. We consider:



$$|f_n-f_m|_2^2=int_a^b|f_n(x)-f_m(x)|^2dx$$



$$=int_a^{frac{b-a}2-frac{2^{-n}}{b-a}}|f_n(x)-f_m(x)|^2dx+int_{frac{b-a}2-frac{2^{-n}}{b-a}}^{frac{b-a}2+frac{2^{-n}}{b-a}}|f_n(x)-f_m(x)|^2dx+int_{frac{b-a}2+frac{2^{-n}}{b-a}}^b|f_n(x)-f_m(x)|^2dx$$



$$=0+int_{frac{b-a}2-frac{2^{-n}}{b-a}}^{frac{b-a}2+frac{2^{-n}}{b-a}}|f_n(x)-f_m(x)|^2dx+0$$



The $0$'s for the first and last integrals because we have $f_n(x)-f_m(x)=0-0$ and $f_n(x)-f_m(x)=1-1$ on each respectively. As to the middle integral, here is what I've gotten so far:



$$int_{frac{b-a}2-frac{2^{-n}}{b-a}}^{frac{b-a}2+frac{2^{-n}}{b-a}}|f_n(x)-f_m(x)|^2dx=int_{frac{b-a}2-frac{2^{-n}}{b-a}}^{frac{b-a}2+frac{2^{-n}}{b-a}}|(2x-b-a)(b-a)(2^{n-2}-2^{m-2})|^2dx$$



Based in the example I am following, supposing that $mge n$, I think that I should be able to argue that the whole thing is bound above by $frac{2^{-n}}{b-a}$ which would mean that $|f_n-f_m|_2^2lefrac{2^{-n}}{b-a}to0$ as $ntoinfty$.



I don't quite see how I'm meant to bound it above by $frac{2^{-n}}{b-a}$, as I keep tripping up over applying that $mge n$, leading me to question whether the approach I have employed here is a fruitful one after all.










share|cite|improve this question











$endgroup$




Problem: Show that $(C([a,b]),|{cdot}|_2)$ is not a complete normed vector space.



I have tried to proceed along the lines given in the accepted answer, and subsequent comment, to this question: Example of a non complete normed vector space. Although, I am running into difficulties and wondering if the example really generalises so well when we have a general $[a,b]$. Is the Cauchy sequence I have chosen below a valid one, or have I overlooked a much simpler example to use in this case?



Attempt: In order to show that $(C([a,b]),|{cdot}|_2)$ is not a complete normed vector space we need to show that there is some Cauchy sequence that does not converge to it's limit in $C([a,b])$. In particular, it will suffice to show that our chosen Cauchy sequence converges under $|cdot|_2$ to a discontinuous function.



Consider the following sequence of functions, $(f_n)_{n=1}^inftysubset C([a,b])$, given by,



$$f_n(x)=begin{cases}
0,,text{when},,xin[a,frac{b-a}2-frac{2^{-n}}{b-a}] \[2ex]
1,,text{when},,xin[frac{b-a}2+frac{2^{-n}}{b-a},b] \[2ex]
frac{(2x-b+a)(b-a)-2^{-n+1}}{2^{-n+2}},,text{when},,xin[frac{b-a}2-frac{2^{-n}}{b-a},frac{b-a}2+frac{2^{-n}}{b-a}]
end{cases}
$$



The definition of $f_n(x)$ when $xin[frac{b-a}2-frac{2^{-n}}{b-a},frac{b-a}2+frac{2^{-n}}{b-a}]$ is the linear interpolant over that particular subinterval of $[a,b]$.



I claim that this is a Cauchy sequence. We consider:



$$|f_n-f_m|_2^2=int_a^b|f_n(x)-f_m(x)|^2dx$$



$$=int_a^{frac{b-a}2-frac{2^{-n}}{b-a}}|f_n(x)-f_m(x)|^2dx+int_{frac{b-a}2-frac{2^{-n}}{b-a}}^{frac{b-a}2+frac{2^{-n}}{b-a}}|f_n(x)-f_m(x)|^2dx+int_{frac{b-a}2+frac{2^{-n}}{b-a}}^b|f_n(x)-f_m(x)|^2dx$$



$$=0+int_{frac{b-a}2-frac{2^{-n}}{b-a}}^{frac{b-a}2+frac{2^{-n}}{b-a}}|f_n(x)-f_m(x)|^2dx+0$$



The $0$'s for the first and last integrals because we have $f_n(x)-f_m(x)=0-0$ and $f_n(x)-f_m(x)=1-1$ on each respectively. As to the middle integral, here is what I've gotten so far:



$$int_{frac{b-a}2-frac{2^{-n}}{b-a}}^{frac{b-a}2+frac{2^{-n}}{b-a}}|f_n(x)-f_m(x)|^2dx=int_{frac{b-a}2-frac{2^{-n}}{b-a}}^{frac{b-a}2+frac{2^{-n}}{b-a}}|(2x-b-a)(b-a)(2^{n-2}-2^{m-2})|^2dx$$



Based in the example I am following, supposing that $mge n$, I think that I should be able to argue that the whole thing is bound above by $frac{2^{-n}}{b-a}$ which would mean that $|f_n-f_m|_2^2lefrac{2^{-n}}{b-a}to0$ as $ntoinfty$.



I don't quite see how I'm meant to bound it above by $frac{2^{-n}}{b-a}$, as I keep tripping up over applying that $mge n$, leading me to question whether the approach I have employed here is a fruitful one after all.







real-analysis functional-analysis banach-spaces norm normed-spaces






share|cite|improve this question















share|cite|improve this question













share|cite|improve this question




share|cite|improve this question








edited Dec 4 '18 at 10:37









Masacroso

13k41746




13k41746










asked Mar 14 '18 at 22:20









Jeremy Jeffrey JamesJeremy Jeffrey James

919613




919613












  • $begingroup$
    one of the functions dominates the other, so you just have to subtract the individual integrals. Can you integrate linear function?
    $endgroup$
    – Jorge Fernández
    Mar 14 '18 at 22:29








  • 1




    $begingroup$
    instead of $frac{b-a}{2}$ you should have $frac{b+a}{2}$
    $endgroup$
    – Orest Bucicovschi
    Mar 15 '18 at 1:58


















  • $begingroup$
    one of the functions dominates the other, so you just have to subtract the individual integrals. Can you integrate linear function?
    $endgroup$
    – Jorge Fernández
    Mar 14 '18 at 22:29








  • 1




    $begingroup$
    instead of $frac{b-a}{2}$ you should have $frac{b+a}{2}$
    $endgroup$
    – Orest Bucicovschi
    Mar 15 '18 at 1:58
















$begingroup$
one of the functions dominates the other, so you just have to subtract the individual integrals. Can you integrate linear function?
$endgroup$
– Jorge Fernández
Mar 14 '18 at 22:29






$begingroup$
one of the functions dominates the other, so you just have to subtract the individual integrals. Can you integrate linear function?
$endgroup$
– Jorge Fernández
Mar 14 '18 at 22:29






1




1




$begingroup$
instead of $frac{b-a}{2}$ you should have $frac{b+a}{2}$
$endgroup$
– Orest Bucicovschi
Mar 15 '18 at 1:58




$begingroup$
instead of $frac{b-a}{2}$ you should have $frac{b+a}{2}$
$endgroup$
– Orest Bucicovschi
Mar 15 '18 at 1:58










1 Answer
1






active

oldest

votes


















2












$begingroup$

Use the mean value theorem:




$$int_a^b f(x),dx = (b-a)f(c), text{ for some } cin langle a , brangle$$




In particular, since your $f_n(x) in [0,1]$, we have:



begin{align}
int_{frac{b-a}2-frac{2^{-n}}{b-a}}^{frac{b-a}2+frac{2^{-n}}{b-a}}|f_n(x)-f_m(x)|^2,dx &= left(frac{b-a}2+frac{2^{-n}}{b-a} - frac{b-a}2+frac{2^{-n}}{b-a} right)underbrace{|f_n(c) - f_m(c)|^2}_{le 2^2 = 4}\
&le 8cdot frac{2^{-n}}{b-a} xrightarrow{ntoinfty} 0
end{align}



Therefore, your sequence is Cauchy.






share|cite|improve this answer









$endgroup$













  • $begingroup$
    I would never have thought to use the MVT, thanks. I guess it's the exact same procedure for showing that $f_nto f$ as $ntoinfty$ where $f$ is the step function on $[a,b]$? In fact, one gets, exactly as you did here, a bound above of $8cdot frac{2^{-n}}{b-a} xrightarrow{ntoinfty} 0$ - is this correct?
    $endgroup$
    – Jeremy Jeffrey James
    Mar 15 '18 at 13:22











Your Answer





StackExchange.ifUsing("editor", function () {
return StackExchange.using("mathjaxEditing", function () {
StackExchange.MarkdownEditor.creationCallbacks.add(function (editor, postfix) {
StackExchange.mathjaxEditing.prepareWmdForMathJax(editor, postfix, [["$", "$"], ["\\(","\\)"]]);
});
});
}, "mathjax-editing");

StackExchange.ready(function() {
var channelOptions = {
tags: "".split(" "),
id: "69"
};
initTagRenderer("".split(" "), "".split(" "), channelOptions);

StackExchange.using("externalEditor", function() {
// Have to fire editor after snippets, if snippets enabled
if (StackExchange.settings.snippets.snippetsEnabled) {
StackExchange.using("snippets", function() {
createEditor();
});
}
else {
createEditor();
}
});

function createEditor() {
StackExchange.prepareEditor({
heartbeatType: 'answer',
autoActivateHeartbeat: false,
convertImagesToLinks: true,
noModals: true,
showLowRepImageUploadWarning: true,
reputationToPostImages: 10,
bindNavPrevention: true,
postfix: "",
imageUploader: {
brandingHtml: "Powered by u003ca class="icon-imgur-white" href="https://imgur.com/"u003eu003c/au003e",
contentPolicyHtml: "User contributions licensed under u003ca href="https://creativecommons.org/licenses/by-sa/3.0/"u003ecc by-sa 3.0 with attribution requiredu003c/au003e u003ca href="https://stackoverflow.com/legal/content-policy"u003e(content policy)u003c/au003e",
allowUrls: true
},
noCode: true, onDemand: true,
discardSelector: ".discard-answer"
,immediatelyShowMarkdownHelp:true
});


}
});














draft saved

draft discarded


















StackExchange.ready(
function () {
StackExchange.openid.initPostLogin('.new-post-login', 'https%3a%2f%2fmath.stackexchange.com%2fquestions%2f2691510%2fshow-that-ca-b-cdot-2-is-not-a-complete-normed-vector-space%23new-answer', 'question_page');
}
);

Post as a guest















Required, but never shown

























1 Answer
1






active

oldest

votes








1 Answer
1






active

oldest

votes









active

oldest

votes






active

oldest

votes









2












$begingroup$

Use the mean value theorem:




$$int_a^b f(x),dx = (b-a)f(c), text{ for some } cin langle a , brangle$$




In particular, since your $f_n(x) in [0,1]$, we have:



begin{align}
int_{frac{b-a}2-frac{2^{-n}}{b-a}}^{frac{b-a}2+frac{2^{-n}}{b-a}}|f_n(x)-f_m(x)|^2,dx &= left(frac{b-a}2+frac{2^{-n}}{b-a} - frac{b-a}2+frac{2^{-n}}{b-a} right)underbrace{|f_n(c) - f_m(c)|^2}_{le 2^2 = 4}\
&le 8cdot frac{2^{-n}}{b-a} xrightarrow{ntoinfty} 0
end{align}



Therefore, your sequence is Cauchy.






share|cite|improve this answer









$endgroup$













  • $begingroup$
    I would never have thought to use the MVT, thanks. I guess it's the exact same procedure for showing that $f_nto f$ as $ntoinfty$ where $f$ is the step function on $[a,b]$? In fact, one gets, exactly as you did here, a bound above of $8cdot frac{2^{-n}}{b-a} xrightarrow{ntoinfty} 0$ - is this correct?
    $endgroup$
    – Jeremy Jeffrey James
    Mar 15 '18 at 13:22
















2












$begingroup$

Use the mean value theorem:




$$int_a^b f(x),dx = (b-a)f(c), text{ for some } cin langle a , brangle$$




In particular, since your $f_n(x) in [0,1]$, we have:



begin{align}
int_{frac{b-a}2-frac{2^{-n}}{b-a}}^{frac{b-a}2+frac{2^{-n}}{b-a}}|f_n(x)-f_m(x)|^2,dx &= left(frac{b-a}2+frac{2^{-n}}{b-a} - frac{b-a}2+frac{2^{-n}}{b-a} right)underbrace{|f_n(c) - f_m(c)|^2}_{le 2^2 = 4}\
&le 8cdot frac{2^{-n}}{b-a} xrightarrow{ntoinfty} 0
end{align}



Therefore, your sequence is Cauchy.






share|cite|improve this answer









$endgroup$













  • $begingroup$
    I would never have thought to use the MVT, thanks. I guess it's the exact same procedure for showing that $f_nto f$ as $ntoinfty$ where $f$ is the step function on $[a,b]$? In fact, one gets, exactly as you did here, a bound above of $8cdot frac{2^{-n}}{b-a} xrightarrow{ntoinfty} 0$ - is this correct?
    $endgroup$
    – Jeremy Jeffrey James
    Mar 15 '18 at 13:22














2












2








2





$begingroup$

Use the mean value theorem:




$$int_a^b f(x),dx = (b-a)f(c), text{ for some } cin langle a , brangle$$




In particular, since your $f_n(x) in [0,1]$, we have:



begin{align}
int_{frac{b-a}2-frac{2^{-n}}{b-a}}^{frac{b-a}2+frac{2^{-n}}{b-a}}|f_n(x)-f_m(x)|^2,dx &= left(frac{b-a}2+frac{2^{-n}}{b-a} - frac{b-a}2+frac{2^{-n}}{b-a} right)underbrace{|f_n(c) - f_m(c)|^2}_{le 2^2 = 4}\
&le 8cdot frac{2^{-n}}{b-a} xrightarrow{ntoinfty} 0
end{align}



Therefore, your sequence is Cauchy.






share|cite|improve this answer









$endgroup$



Use the mean value theorem:




$$int_a^b f(x),dx = (b-a)f(c), text{ for some } cin langle a , brangle$$




In particular, since your $f_n(x) in [0,1]$, we have:



begin{align}
int_{frac{b-a}2-frac{2^{-n}}{b-a}}^{frac{b-a}2+frac{2^{-n}}{b-a}}|f_n(x)-f_m(x)|^2,dx &= left(frac{b-a}2+frac{2^{-n}}{b-a} - frac{b-a}2+frac{2^{-n}}{b-a} right)underbrace{|f_n(c) - f_m(c)|^2}_{le 2^2 = 4}\
&le 8cdot frac{2^{-n}}{b-a} xrightarrow{ntoinfty} 0
end{align}



Therefore, your sequence is Cauchy.







share|cite|improve this answer












share|cite|improve this answer



share|cite|improve this answer










answered Mar 14 '18 at 22:35









mechanodroidmechanodroid

27.1k62446




27.1k62446












  • $begingroup$
    I would never have thought to use the MVT, thanks. I guess it's the exact same procedure for showing that $f_nto f$ as $ntoinfty$ where $f$ is the step function on $[a,b]$? In fact, one gets, exactly as you did here, a bound above of $8cdot frac{2^{-n}}{b-a} xrightarrow{ntoinfty} 0$ - is this correct?
    $endgroup$
    – Jeremy Jeffrey James
    Mar 15 '18 at 13:22


















  • $begingroup$
    I would never have thought to use the MVT, thanks. I guess it's the exact same procedure for showing that $f_nto f$ as $ntoinfty$ where $f$ is the step function on $[a,b]$? In fact, one gets, exactly as you did here, a bound above of $8cdot frac{2^{-n}}{b-a} xrightarrow{ntoinfty} 0$ - is this correct?
    $endgroup$
    – Jeremy Jeffrey James
    Mar 15 '18 at 13:22
















$begingroup$
I would never have thought to use the MVT, thanks. I guess it's the exact same procedure for showing that $f_nto f$ as $ntoinfty$ where $f$ is the step function on $[a,b]$? In fact, one gets, exactly as you did here, a bound above of $8cdot frac{2^{-n}}{b-a} xrightarrow{ntoinfty} 0$ - is this correct?
$endgroup$
– Jeremy Jeffrey James
Mar 15 '18 at 13:22




$begingroup$
I would never have thought to use the MVT, thanks. I guess it's the exact same procedure for showing that $f_nto f$ as $ntoinfty$ where $f$ is the step function on $[a,b]$? In fact, one gets, exactly as you did here, a bound above of $8cdot frac{2^{-n}}{b-a} xrightarrow{ntoinfty} 0$ - is this correct?
$endgroup$
– Jeremy Jeffrey James
Mar 15 '18 at 13:22


















draft saved

draft discarded




















































Thanks for contributing an answer to Mathematics Stack Exchange!


  • Please be sure to answer the question. Provide details and share your research!

But avoid



  • Asking for help, clarification, or responding to other answers.

  • Making statements based on opinion; back them up with references or personal experience.


Use MathJax to format equations. MathJax reference.


To learn more, see our tips on writing great answers.




draft saved


draft discarded














StackExchange.ready(
function () {
StackExchange.openid.initPostLogin('.new-post-login', 'https%3a%2f%2fmath.stackexchange.com%2fquestions%2f2691510%2fshow-that-ca-b-cdot-2-is-not-a-complete-normed-vector-space%23new-answer', 'question_page');
}
);

Post as a guest















Required, but never shown





















































Required, but never shown














Required, but never shown












Required, but never shown







Required, but never shown

































Required, but never shown














Required, but never shown












Required, but never shown







Required, but never shown







Popular posts from this blog

Quarter-circle Tiles

build a pushdown automaton that recognizes the reverse language of a given pushdown automaton?

Mont Emei